9/5-MBE TORTS

¡Supera tus tareas y exámenes ahora con Quizwiz!

Question 1371 In what began as consensual roughhousing on a public sidewalk, a high school freshman refused to stop despite a request by the other, much larger participant, a college senior. As a consequence, the senior began to pummel the freshman. The freshman managed to escape from the senior, and fled with the senior in pursuit. A deliveryman witnessed only the freshman's flight from the senior. As the freshman neared the deliveryman, the senior called out to the deliveryman, "Stop that boy!" The deliveryman, responding to the senior's appeal, used reasonable force to trip the freshman as he came running by the deliveryman. As a result, the freshman suffered physical injuries. The freshman brought suit against the deliveryman to recover damages for his injuries. Who will prevail? Answers: The freshman, because the deliveryman purposefully caused the freshman's injuries. The freshman, because the deliveryman was an intermeddler. The deliveryman, because he used reasonable force in responding to the senior's request. The deliveryman, because the freshman was the initial aggressor.

Answer choice A is correct because the deliveryman committed a battery. His intentional tripping of the freshman caused the freshman's injuries. Answer choice B is incorrect because, although the deliveryman did interpose himself into the students' affair, an intermeddler may nevertheless escape liability when the defense of others applies. Answer choice C is incorrect because, although the deliveryman used reasonable force in tripping the freshman, the deliveryman could not avail himself of the defense of others. The deliveryman could not step into the shoes of the senior because the senior, by chasing the freshman, was not acting in self-defense but instead was retaliating. In addition, the deliveryman could not assert that he had a reasonable belief that the senior was acting in self-defense, because the senior was the pursuer rather than the pursued. Answer choice D is incorrect because, although the freshman was the initial aggressor by continuing to roughhouse after the senior withdrew his consent, the senior became the aggressor by chasing the freshman, thereby acting in retaliation rather than self-defense.

Question 4260 A business was founded by two partners. As the business grew, the two partners became increasingly busy. The partnership hired a limousine service, which provided a car and driver, to allow the partners to work en route to meetings. Both partners were on their way to a meeting when the limousine driver negligently ran a red light and crashed the car. One of the partners was not wearing a seat belt and suffered serious injury. The other partner was wearing a seat belt and did not suffer any injuries. The injured partner was unable to work for weeks due to his injuries, resulting in serious disruptions to the business. The partnership sued the limousine service on a negligence theory, alleging that it suffered significant losses due to the absence of the injured partner. The applicable state law recognizes a partnership as a separate legal entity that can sue and be sued. Is the partnership likely to recover in its negligence suit? Answers: No, because the business suffered only economic losses, with no personal or property damage. No, because the business may recover only from the driver, not his employer. Yes, because the driver's negligence caused the business to suffer actual damages. Yes, because the driver owed a duty of care to the business and the individual partners.

Answer choice A is correct. A negligence plaintiff must prove actual injury. A plaintiff who suffers only economic loss without any related personal injury or property damage cannot recover such loss through a negligence action. In this case, the business suffered only economic losses, and thus may not recover. Answer choice B is incorrect because the business may not recover from either the driver or his employer. Moreover, an employer is liable for the negligent actions of its employee committed in the scope of employment. Answer choice C is incorrect because the business suffered only economic loss, not actual damages. Answer choice D is incorrect because, even if the driver owed a duty of care to the business, the business cannot satisfy the elements of a negligence action because it cannot prove damages.

Question 7119 A woman decided to take up beekeeping as a hobby. She purchased some beehive boxes, worker bees, and one queen bee, and began making honey in her backyard bee farm. The woman's bees did often leave the beehive boxes and occasionally ventured onto neighboring properties, but there were still fewer bees than one might encounter near a typical flower bush or in a park. A man that lived next door to the woman was extremely allergic to bee stings, and if stung, would immediately have to go to the hospital. Because of his extreme allergy, the man rarely left the house. When the man saw the beehive boxes in the woman's backyard, he sued her for private nuisance. Is the man likely to prevail? Answers: No, because the average person would not be inconvenienced by the beehive boxes. No, because the man rarely left his home due to the presence of the bees. Yes, because the man had an extreme allergic reaction to bee stings. Yes, because the beehive boxes were an unreasonable interference.

Answer choice A is correct. A private nuisance is a thing or activity that substantially and unreasonably interferes with another individual's use or enjoyment of his land. A person with special sensitivities can recover only if the average person would be offended, inconvenienced, or annoyed. Here, since the beehives housed fewer bees than one might encounter near a typical flower bush or in a park, the average person would be unlikely to be inconvenienced, annoyed, or offended by the bees. Answer choice B is incorrect because even if the man rarely leaves his home, he has a possessory interest in his property. This would make him a proper plaintiff if there had also been a substantial interference that would be offensive, inconvenient, or annoying to a normal, reasonable person in the community. Answer choice C is incorrect because the man's sensitivity to bee stings does not change the standard that must be met to establish a substantial interference for a private nuisance tort. Answer choice D is incorrect. Here, the interference was not unreasonable because raising bees and making honey is a useful hobby, and outweighs the minor issue of a few bees occasionally visiting neighboring properties.

Question 5978 A plastics manufacturer opened a plant in a rural area. As a byproduct of the manufacturing, the plant emitted a harmless gas. The gas was odorless to all but a few individuals with a rare olfactory disorder. A few years after the plant opened, a man purchased a house near the plant. Soon after, the man developed the rare olfactory disorder. The odor of the gas caused the man to suffer from headaches and nausea. The man brought an action against the plastics manufacturer for nuisance, seeking to enjoin the plant's activity. Will the man's action be successful? Answers: No, because the gas emissions would not be considered offensive to an average person in the community. No, because the man moved to the area after the plant was already in operation. Yes, because the man suffered harm different in kind from that of the public at large. Yes, because the gas emissions substantially interfered with the man's use and enjoyment of his home.

Answer choice A is correct. A private nuisance is a thing or activity that substantially and unreasonably interferes with another individual's use or enjoyment of his land. A substantial interference is one that would be offensive, inconvenient, or annoying to a normal, reasonable person in the community. A person with special sensitivities can recover only if the average person would be offended, inconvenienced, or annoyed. In this case, only persons with rare olfactory conditions could smell the otherwise harmless gas emitted by the plant. Because the gas emissions would not be offensive to a normal, reasonable person in the community, the interference is not substantial and thus the man's action will fail. Answer choice B is incorrect. Although the fact that the man "came to the nuisance" is relevant and is evidence that the jury may consider, it does not automatically preclude the man from recovering under a theory of private nuisance. Answer choice C is incorrect because it states one of the requirements for a public nuisance cause of action. Here, the claim is for a private nuisance because the public at large does not suffer ill effects from the gas emitted by the manufacturer. Answer choice D is incorrect. Although the gas emissions did significantly interfere with the man's use and enjoyment of his land, they are not offensive to a normal, reasonable person in the community. Thus, the man cannot recover in a private nuisance action.

Question 6651 A newly retired chemist bought a house in a small but densely populated rural town. The chemist still enjoyed conducting experiments in her spare time. Because she used very strong chemicals, she always experimented with her windows open to keep fresh air circulating in her home. A computer programmer, who lived a few doors down on the same road as the chemist, had lived in the neighborhood for 15 years. The programmer worked from home, and because he had many computers and other hardware that generated a lot of heat, he also kept his windows open to keep the house cooler. Since birth, the programmer had suffered from a rare disease known as anosmia, which prevented him from smelling anything. As a result, the programmer could not smell the noxious, though ultimately harmless, fumes emanating from the chemist's home, but he knew of them based upon community gossip. Setting aside the issue of damages, should the programmer prevail in a private nuisance action against the chemist? Answers: Yes, because other neighbors in the community were able to smell the fumes coming from the chemist's home. Yes, because the programmer suffered harm that is different from that suffered by other community members. No, because the programmer cannot smell the noxious odors emanating from the chemist's home. No, because he assumed the risk of encountering the malodorous smells by keeping his windows open.

Answer choice A is correct. A private nuisance is a thing or activity that substantially and unreasonably interferes with another individual's use or enjoyment of his land. The interference must be intentional, negligent, reckless, or the result of abnormally dangerous conduct to constitute nuisance. Finally, a substantial interference is one that would be offensive, inconvenient, or annoying to a normal, reasonable person in the community. The chemist's intentional experiments created smells that substantially and unreasonably interfered with the programmer's enjoyment of his home. Because it was annoying to other, presumably reasonable, neighbors in the community, the programmer can prevail as well under this theory. Answer choice B is incorrect because it states the standard for a private citizen's claim for public nuisance. Answer choice C is incorrect because it does not matter if the programmer cannot smell the fumes—it only matters if the average person would be offended, inconvenienced, or annoyed. However, the amount of damages the programmer recovers may be affected. Finally, answer choice D is not the best answer because although the programmer voluntarily and knowingly left his windows open even though he knew about the fumes, there was nothing inherently dangerous about leaving his windows open during the day so this would not be a strong defense to his recovery.

Question 6654 A successful politician suffered from chronic social anxiety that caused him to have debilitating panic attacks before any public speaking event. The politician was embarking on an extensive campaign tour across the country because it was an election year, so he asked his doctor about possible medications to alleviate his social anxiety. The doctor recommended a popular prescription drug called CalmX, which had successfully helped many of the doctor's patients to overcome their anxiety disorders. The makers of CalmX warned all doctors prescribing the drug of all problems associated with CalmX. One side effect—extreme flatulence—could occur, but only when combined with the excessive consumption of a very rare imported cheese. Due to that side effect's rarity, the CalmX manufacturers did not warn consumers about it in the instructions that accompanied the drug. The politician successfully took CalmX for much of his campaign tour, but he combined it with the excessive consumption of the rare imported cheese on the day of a critical televised debate. During the debate, the politician experienced extreme flatulence. The video of the debate went viral and caused him to lose the election. The politician brought a strict products liability action against the CalmX manufacturers for failure to warn about the drug's side effect; will he prevail? Answers: No, because the CalmX manufacturers informed the doctor about the side effects associated with the drug. No, because the consumers that might suffer from this particular side effect of CalmX were very few in number. Yes, because the CalmX manufacturers did not warn consumers about all of the side effects associated with CalmX. Yes, because the CalmX manufacturers are commercial suppliers, so they are liable for harm caused by the drug.

Answer choice A is correct. According to the "learned intermediary rule," the manufacturer of a prescription drug typically satisfies its duty to warn the consumer by informing the prescribing physician of problems with the drug rather than informing the patient taking the drug. For this reason answer choice C is incorrect. Answer choice B is incorrect because although the number of consumers that might suffer from the side effect of CalmX was limited, the CalmX manufacturers could still be liable for failure to warn if the failure rendered the product not reasonably safe. Answer choice D is incorrect because in order to be liable as a commercial supplier in a strict products liability action, the product itself must also be defective. In this case, the politician's strict products liability action was based upon the CalmX manufacturers' failure to warn, but as discussed above, this duty was satisfied when they informed the doctor of the potential side effect of CalmX.

Question 4263 A man managed a small hotel. His ex-wife stopped at the hotel one day, demanding that the he give her overdue child support. The hotel manager asked his ex-wife to leave because he did not wish to discuss their financial problems while he was at work. On her way out, she let herself into the manager's office with a key she had retained, unbeknownst to the manager, to use his private bathroom. The sink in the bathroom had stopped up and flooded the bathroom floor. Because she was upset and rushed, the ex-wife did not notice that the floor was wet, and she slipped and sprained her ankle. The ex-wife sued the hotel for negligence. The evidence at trial showed that neither the manager nor the other employees of the hotel knew that the bathroom had flooded. Is the ex-wife likely to prevail on her negligence claim? Answers: No, because the hotel did not owe the ex-wife, an undiscovered trespasser, an affirmative duty of care. No, because the hotel's only duty was to warn or protect the ex-wife from concealed dangerous artificial conditions that involved risk of death or serious bodily harm. Yes, because the hotel owed a duty to conduct reasonable inspections of the property and make it safe for the protection of the ex-wife. Yes, because an innkeeper is held to the highest standard of care consistent with the practical operation of the business.

Answer choice A is correct. Generally, landowners owe no affirmative duty to undiscovered trespassers. Even though the ex-wife was not initially an undiscovered trespasser, she became one when she sneaked into the manager's office, an area that was locked and off-limits to the public. Accordingly, the hotel owed no affirmative duty to her and is not liable for her injuries. Answer choice B is incorrect because it states the standard for discovered trespassers. The ex-wife may have initially been considered a discovered trespasser, as she was not at the hotel for a business purpose and was asked to leave. However, once she sneaked into the manager's locked office, she became an unknown trespasser. In any event, the wet floor was not a concealed danger, just because the ex-wife did not notice it. Answer choice C is incorrect because it describes the duty owed to an invitee, or one who enters the land of another by invitation, often for the business purposes of the landowner. The ex-wife was not an invitee, since she was not invited onto the property, was asked to leave, and entered an employee-only area of the hotel. Answer choice D is incorrect because, although common carriers and innkeepers are held to the highest duty of care consistent with the practical operation of the business, this duty applies only to customers and guests.

Question 2985 Ten treadmills line the windows in a designated area of a popular gym. Eight of the treadmills are modern and fully functional, while two of them periodically stop for unknown reasons. No runner has ever been injured as a result of any fall on the two treadmills. The gym had originally posted warnings on the two treadmills indicating the nature of the problem, but after realizing the high demand for the extra two treadmills, management removed the warnings and decided to keep the malfunctioning treadmills until they were able to purchase replacements. One day, one of the treadmills stopped abruptly. The runner using the treadmill had been reaching down to retrieve his dropped headphones, and the sudden stop caused him to pitch forward, breaking his outstretched hand. Is the gym liable for the damage resulting from the runner's broken hand? Answers: Yes, because the runner's broken hand was a foreseeable injury. Yes, because the gym owned the treadmill. No, because the injury was not caused by the malfunction. No, because the runner was not using the treadmill properly.

Answer choice A is correct. If a plaintiff's injury is a foreseeable result of a defendant's negligence, then the defendant will be liable for the injury. Here, a runner's bending down while on the treadmill may not be foreseeable, but an injury associated with the stopping of the treadmill is certainly foreseeable. Answer choice B is incorrect because the ownership of the treadmill is not the only factor that determines liability. Answer choice C is incorrect because, although the exact injury that ultimately occurred in this case might not have been foreseeable, it was foreseeable that a runner on a malfunctioning treadmill would be injured if the treadmill stopped abruptly. Answer choice D is incorrect because, while contributory negligence might bar recovery in certain jurisdictions, the plaintiff's action would likely not constitute negligence. Even if it did, negligence is a foreseeable intervening act, so the runner's bending down would not break the chain of causation between the gym's negligence and the runner's injury.

Question 4297 A man and his friend decided to drive downstate to watch the homecoming football game at their alma mater. The man offered to drive and refused any compensation from the friend. On the way, the man negligently ran a stoplight and collided with another vehicle. The friend was severely injured. The friend sued the man for injuries sustained in the accident. The applicable jurisdiction recently enacted a guest statute with regard to automobile passengers. Will the friend prevail in her suit against the man? Answers: No, because the man was merely negligent. No, because a driver is liable only for injuries caused to passengers that have provided monetary compensation. Yes, because the man was strictly liable for injuries he caused to any passenger, including a guest. Yes, because the man owed his friend a duty to exercise ordinary care.

Answer choice A is correct. In most jurisdictions, an automobile driver owes a duty of ordinary reasonable care to all passengers, including guests (i.e., individuals who do not confer an economic benefit for the ride on the driver). However, a minority of states distinguish between the two with "guest statutes," which impose only a duty to refrain from gross or wanton and willful misconduct with respect to a guest. Here, the driver was merely negligent, and therefore the passenger would not be able to recover in a guest statute jurisdiction. Answer choice B is incorrect because a guest statute does not absolve the driver of liability to a non-paying passenger, but requires such passengers to establish a higher level of culpability by the driver in order to recover. Answer choice C is incorrect because a guest statute does not impose strict liability on the driver of a vehicle with respect to his passengers. Answer choice D is incorrect because, although most jurisdictions impose upon drivers a duty to exercise ordinary care with respect to guests as well as passengers, those that have guest statutes impose only a duty to refrain from gross or wanton and willful misconduct with a guest in the car.

Question 7213 After leaving a bar late one evening, an intoxicated college student headed home on his bicycle. Halfway there, he lost his balance and crashed. When he fell, his head hit the curb and he lost consciousness. A nearby driver was unknowingly driving through an intersection with a four-way stop; the stop signs had been pulled down earlier that night. As the driver went through the intersection, he noticed the unconscious student lying in the road. He stepped on the brakes, but the brakes failed completely, and the driver ran over the student's legs. The student sued the driver for negligence to recover damages for his broken legs. At trial, it was conclusively established that the driver had brought his car to a mechanic to be serviced earlier on the day of the accident, and that the mechanic's negligent maintenance had caused the brakes to fail. It was also established that if the brakes had been serviced properly, the driver would have been able to stop his car before injuring the student. Which of the following is the driver's strongest defense? Answers: The driver used reasonable care in the maintenance of his car. The driver was reasonably ignorant of the four-way stop. The student assumed the risk of injury by riding a bike while intoxicated. The student's injuries were not proximately caused by the driver's running the stop sign.

Answer choice A is correct. In order to recover in a negligence action, the plaintiff must establish that the defendant owed the plaintiff a duty, which the defendant breached, that the defendant's actions were the actual and legal cause of the injury, and that the plaintiff suffered damages. A breach of duty occurs when the defendant departs from the conduct expected of a reasonably prudent person acting under similar circumstances. An automobile driver has a duty to make sure that the automobile is kept in good repair. Here, if the driver used reasonable care in the maintenance of his brakes, he would not have breached his duty of care. Hence, he could not be found negligent. Therefore, this is the driver's strongest argument. Answer choice B is incorrect. This is not a case of negligence per se, because failing to stop at the intersection was not the proximate cause of the student's injuries. Accordingly, arguing any defenses to negligence per se would not be the driver's strongest argument. Answer choice C is incorrect. Although the student may have assumed the risk of falling, he did not assume the risk that another driver would run over his legs by riding a bike while intoxicated. Answer choice D is incorrect. Although this argument is a strong defense against a negligence per se theory of recovery, answer choice A is a stronger defense because it also applies to a general negligence action.

Question 7208 A landowner who lived on a steep hill hired a landscaper to build a retaining wall behind his home so that he could have a level back yard. After the landscaper completed the work, the landowner installed a small koi pond in his newly leveled back yard. However, a week after the pond was installed, all of the water in the pond seeped out and caused the retaining wall to collapse outward onto an adjoining neighbor's property. The collapse severely damaged the neighbor's fence and flower garden. The neighbor sued the landowner to recover for this damage. The court awarded the neighbor damages based on vicarious liability and the landowner's negligent installation of the koi pond. Can the landowner bring a successful suit for damages against the landscaper? Answers: Yes, because the neighbor recovered a judgment based on vicarious liability. Yes, because the landscaper's breach of duty to the landowner caused damages. No, because there was no indemnification agreement between the landowner and the landscaper. No, because the landowner's negligent installation of the koi pond contributed to his liability.

Answer choice A is correct. Indemnification generally applies when one person is vicariously liable for the other's wrongdoing. Here, because the facts provide that the neighbor's judgment relied at least in part on a finding of vicarious liability, the landowner can probably seek indemnity from the landscaper. Answer choice B is incorrect because, based on these facts, it is unclear whether the landscaper breached a duty to the landowner. The court has found that the landowner was negligent when he installed the koi pond, and that he was vicariously liable for negligence committed by the landscaper. However, this finding only means that the landscaper has breached a duty to the neighbor, since the neighbor was the plaintiff in that action. Additionally, the only loss the landowner has suffered is an adverse judgment, and a plaintiff cannot recover economic damage in a negligence action without any related personal or property damage. On these facts, it is not clear that the landowner has suffered any damages that would support a claim of tortious negligence against the landscaper. Answer choice C is incorrect because no indemnification agreement is required to allow indemnification when a party has been held vicariously liable for the negligence of another. Answer choice D is incorrect because the landowner's negligence with respect to his neighbor will not bar him from seeking indemnification for the portion of his judgment based on vicarious liability.

Question 2959 A patient visited his physician to get an influenza vaccine. Before seeing his physician, the patient completed a form that, among other things, asked the patient to list his allergies. The patient, believing that the question referenced allergies to medication, listed the medications to which he is allergic. He did not list his food allergies, which include eggs. Before administering the vaccine, the physician had a discussion with the patient regarding various side effects associated with the vaccine. They did not specifically discuss the patient's allergies. The physician administered the vaccine, which is made with eggs. The patient had a severe allergic reaction, and died shortly thereafter. Would the physician be liable in a negligence action for the patient's death? Answers: Yes, because the physician did not obtain informed consent. Yes, because the physician did not exhibit the same skill, knowledge, and care of a doctor of similar education and experience. No, because the physician exhibited the same skill, knowledge, and care as another physician in the community. No, because the physician acted in a reasonably prudent manner.

Answer choice A is correct. Negligence is the commission of an act (or the failure to act), without wrongful intent, that falls below the minimum degree of ordinary care imposed by law to protect others against unreasonable risk of harm. Physicians are under a specific obligation to explain the risks of a medical procedure to a patient in advance of a patient's decision to consent to treatment. Failure to comply with this "informed consent" doctrine constitutes a breach of the physician's duty owed to the patient and is actionable as medical malpractice (medical negligence). By failing to discuss the patient's allergies, and the risk of death from the eggs in the vaccine, the doctor breached his duty to the patient to obtain informed consent. Answer choice B is incorrect because it basically describes the standard of care imposed upon children rather than professionals (though substituting "education" for "age"). Answer choice C is incorrect because, although it lists the correct standard for professionals, it fails to account for a physician's duty to obtain informed consent. Answer choice D is incorrect because a physician is under a special duty to obtain informed consent, which the physician failed to do in this case.

Question 6526 An 80-year-old woman who had been a zoologist for most of her life suffered from Alzheimer's disease. Due to changes in her brain chemistry caused by the disease, the woman often had false perceptions of objects. One day, a guest at the woman's senior citizen home approached the woman while wearing a furry coat. The furry coat worn by the guest caused the woman to hallucinate and believe a lion was about to attack her. To protect herself, the woman hit the guest in the head with her cane. The guest sued the woman for battery. The applicable jurisdiction follows the "single-intent" rule for battery. What is the woman's best defense? Answers: The woman believed that she was hitting a lion with her cane. The woman did not comprehend the wrongfulness of her act. The woman did not intend to harm the guest. The woman was trying to protect herself from the guest.

Answer choice A is correct. One is liable for battery when she intentionally causes a harmful or offensive contact with the person of another and acts with the intent to cause such contact. In this case, because the woman thought that the guest was a lion, she will not be liable for battery because she was not aware that she was striking a person. Answer choice B is incorrect. A mentally ill person may be held liable for a battery if she has the mental capacity to form the intent to strike a person, even if she is unable to understand the wrongfulness of her act. Therefore, this is not an effective defense. Answer choice C is incorrect. Under the "single-intent" rule, a defendant may be liable if he intends merely to bring about the contact. While the contact must be harmful or offensive, the defendant need not intend that result. Thus, there is no requirement that the woman intended to cause harm to the guest, only that she intended to cause the unpermitted contact. Answer choice D is incorrect. The woman believed that the guest was about to attack her. Thus, she would likely be permitted to use reasonable force to defend herself. However, a justified reason or excuse, even if successful, is not as good of a defense as the failure to establish a required element for battery. Therefore, answer choice A is better than answer choice D.

Question 4264 An office worker who suffered from severe asthma was trapped in an elevator at work for over two hours. During that time, she experienced acute anxiety. After her rescue, she was taken to hospital where it was determined that she had suffered permanent damage to her airways, and that the damage would not have occurred had her asthma not been so severe. While the damage would never be fully repaired, expensive, ongoing treatments could improve the worker's condition. The office worker brought a personal injury suit against the owner of the building, seeking to recover compensation for the permanent damage to her airways; medical expenses, both past and future; pain and suffering during the ordeal; and attorney's fees. Which of the following is the office worker unable to recover? Answers: Attorney's fees. Compensation for the permanent damage to her airways. Future medical expenses. Pain and suffering damages.

Answer choice A is correct. The measure of damages in a personal injury action includes all actual damages incurred, past and future pain and suffering (e.g., emotional distress), medical expenses, lost wages and any reduction in future earnings capacity, and loss of consortium. Under the "thin-skull" or "eggshell-plaintiff" rule, the defendant is liable for the full extent of the plaintiff's injuries due to the plaintiff's pre-existing physical or mental condition or vulnerability, even if the extent is unusual or unforeseeable. Attorney's fees in a personal injury suit are not recoverable. In this case, the plaintiff may not obtain attorneys' fees. Answer choice B is incorrect because the fact that the worker's preexisting asthma exacerbated the damage to her airways is irrelevant, as a defendant is liable for the full extent of the plaintiff's injuries, even if increased by the plaintiff's preexisting physical or mental condition. Answer choices C and D are incorrect because both future medical expenses and pain and suffering damages are recoverable in a personal injury negligence action.

Question 5999 The defendant owns an alligator, which she keeps in a pen in her front yard; local laws do not prohibit owning such an animal. The plaintiff was walking by the defendant's yard on the sidewalk when he noticed the alligator. Although the plaintiff was not in actual danger from the alligator, he jumped backwards in fear when he saw the creature, and severely sprained his ankle. The plaintiff sued the defendant to recover damages for injury. Of the following, which provides the defendant with her best argument against this claim? Answers: The alligator was properly restrained when the plaintiff suffered his injury. The plaintiff cannot recover because he was in no actual danger from the alligator. No local law precludes the defendant from owning an alligator. The plaintiff cannot recover because his injury was not caused by the alligator's dangerous propensities.

Answer choice A is correct. The possessor of a wild animal is strictly liable for harm done by that animal, in spite of any precautions the possessor has taken to confine the animal or prevent the harm, if the harm arises from a dangerous propensity that is characteristic of such a wild animal or of which the owner has reason to know. Strict liability also applies to an injury caused by a plaintiff's fearful reaction to the sight of an unrestrained wild animal. In this case, although the defendant owned a wild animal, she is not strictly liable for the injury caused by the plaintiff's fearful reaction to the sight of the alligator because the alligator was not roaming free, but instead was confined within a pen. Consequently, the defendant is not liable to the plaintiff in negligence, nor is she strictly liable, because the alligator was properly confined. Answer B is incorrect because strict liability for harm caused by wild animals extends to harm caused by the reasonable fear people feel in the vicinity of the wild animal, if unrestrained. Answer choice C is incorrect because compliance with a statute, a regulation, or an ordinance generally does not prove the absence of negligence, and would not protect the defendant from strict liability if the animal had escaped. Answer choice D is incorrect because the injury need not be caused by the alligator's dangerous propensities; an injury caused by the plaintiff's fearful reaction to an unrestrained wild animal is sufficient.

Question 6159 A pool cleaner at a country club used chlorine purchased from a company to kill the harmful bacteria that grew in his customers' pools. The company sold its chlorine in plastic containers with plastic screw tops. Despite knowing that the containers may leak if not stored upright, the pool cleaner haphazardly stacked the containers on their sides and stored them in a shed. The shed also contained other maintenance equipment for the country club, and a maintenance worker assigned to organize the shed sustained injuries after inhaling excess fumes that had built up in the small shed from the leaking containers. The containers had warnings that said, simply, "Caution: Chlorine," but did not state specifically that chlorine fumes could build up if chlorine leaked in a hot, unventilated space, or that such fumes were harmful if inhaled. The maintenance worker filed suit against the company under a theory of strict products liability. If the company wins, which of the below is the most likely reason for its success? Answers: There was no economically feasible reasonable alternative design for the containers. The maintenance worker was not the appropriate plaintiff to bring this action. The pool cleaner's negligence contributed to the maintenance worker's injury. The "Caution: Chlorine" label was an insufficient warning as to the risk of harmful fumes.

Answer choice A is correct. Using the risk-utility test, to prevail on a claim under a strict products liability design defect theory, a jury must determine whether the risks posed by a product outweigh its benefits. To succeed, a plaintiff must prove that a reasonable alternative design was available to the defendant and the failure to use that design has rendered the product not reasonably safe. The reasonable alternative design must be economically feasible. Here, if the finder of fact determined that there was no economically feasible alternative design, the plaintiff could not prevail under a strict liability theory. Answer choice B is incorrect because the commercial manufacturer, distributor, retailer, or seller of a product owes a duty of reasonable care to any foreseeable plaintiff; it is not necessary that the plaintiff be the purchaser of the defective product. Answer choice C is incorrect because while the plaintiff's own negligence can reduce the plaintiff's recovery under a strict liability claim, here, there is no evidence that the plaintiff, the maintenance worker, was negligent. The contributory negligence of a third party would not affect the defendant's own liability for the product it manufactures. Answer choice D is incorrect because, if true, this would be a fact in favor of the plaintiff, not the defendant. It would make the defendant liable under the failure to warn theory of strict liability.

Question 1332 The owner of pastureland permits a herder to keep several goats in the pasture. The goats wander off the land, leap over a fence properly maintained by a neighbor, and ramble into the neighbor's garden. Once in the garden, the goats eat the vegetables growing there. In a strict liability action brought by the neighbor against the landowner and the herder, who is liable? Answers: Only the landowner. Only the herder. Both the landowner and the herder. Neither the landowner nor the herder.

Answer choice B is correct because the owner of any animal, wild or domestic (other than a household pets) is strictly liable for any reasonably foreseeable damages caused by the animal while trespassing on another's land. Strict liability does not extend to the owner of the land on which the animals are kept, even when the animals are on the land with the landowner's permission, unless the landowner also has the right to possess the animals. Consequently, answer choices A, C, and D are incorrect.

Question 6546 A man was barbecuing a whole chicken on a charcoal grill in his backyard. Inexperienced at grilling, he used too much lighter fluid on the charcoal and failed to properly clean the grate of excess grease and fat. When the man lit the charcoal, the charcoal and chicken burst into flames. Immediately after, the man's wife entered the backyard. She was afraid that the fire would spread to the ground, which was covered in synthetic lawn turf, and ultimately to their nearby home. Remembering that their neighbor had a pool, the wife rolled the entire grill into the neighbor's backyard and dumped the grill and the chicken into the neighbor's pool. In doing so, she pushed the grill through the neighbor's garden, destroying his crop of organic heirloom tomatoes. The neighbor sued the wife for trespass to land. What can the neighbor recover? Answers: Nothing, due to the doctrine of private necessity. Actual damages only. Actual and nominal damages. Actual, nominal and punitive damages.

Answer choice B is correct. A trespass to land occurs when one's intentional act causes a physical invasion of another's land. A trespasser need only have the intent to enter the land (or to cause a physical invasion), not the intent to commit a wrongful trespass. Here, the wife committed trespass to land because she intentionally entered the neighbor's backyard. The privilege of necessity is available to a person who enters or remains on the land of another (or interferes with another's personal property) to prevent serious harm, which typically is substantially more serious than the invasion or interference itself. Private necessity is a qualified privilege to protect an interest of the defendant or a limited number of other persons from serious harm. The privilege applies if the interference was reasonably necessary to prevent a serious injury from nature or another force not connected with the property owner. Despite this privilege, the property owner is entitled to recover actual damages, but cannot recover nominal or punitive damages nor use force to eject the defendant. In this case, the privilege of private necessity is available to the wife because crushing the tomato plants and contaminating the neighbor's pool is less serious than the danger of her home burning down, and it was reasonably necessary to use the neighbor's pool to put out the fire. Despite the privilege, the neighbor is still entitled to recover actual damages. However, he cannot recover nominal or punitive damages. For this reason, answer choices A, C and D are incorrect.

Question 1490 A professional football player injured his knee during a football game. The player delayed seeking medical attention for his injuries for several days, which aggravated those injuries. When the player finally sought treatment from an orthopedic surgeon, the orthopedic surgeon negligently performed the operation on the player's knee. After surgery, the player failed to follow the surgeon's post-operative instructions. All of the player's actions coupled with the surgeon's negligence contributed to a permanent injury to the player's knee. As a consequence of the injury, the player's ability to continue to play professional football has been adversely affected and the player has suffered emotional distress which has not manifested itself as a physical injury. The player sued the orthopedic surgeon for damages attributable to the surgeon's medical treatment of the player's knee injuries. Assuming that the monetary effect of each of the following can be established with reasonable certainty, which would a fact finder not take into account in determining the amount, if any, of the damages to which the player would otherwise be entitled due to the surgeon's negligence? Answers: The plaintiff's inability to continue playing professional football due to his knee injury. The plaintiff's failure to promptly seek medical care. The plaintiff's emotional distress. The plaintiff's failure to follow the surgeon's post-operative instructions.

Answer choice B is correct. Although the plaintiff's failure to promptly seek medical attention aggravated his injuries, it was the aggravated injury that the defendant agreed to treat. Consequently, the surgeon is not liable for such injuries, but only for the harm caused by the surgeon's negligent treatment of those injuries. (Note: When the plaintiff's initial harm is inflicted by the defendant, rather than by the plaintiff or an independent third party, the plaintiff's failure to promptly seek medical care can constitute a failure to mitigate damages, which can be taken into account in determining the amount of damages to which the plaintiff is entitled.) Answer choice A is incorrect because the loss of future earnings that is attributable to harm that is caused by the defendant can be recovered by a plaintiff, if such loss can be established with reasonable certainty. Answer choice C is incorrect because damages for emotional distress are recoverable in a negligence action when the plaintiff has suffered a physical injury even though the emotional distress itself has not been manifested through a physical injury. Answer choice D is incorrect because a failure by the plaintiff to mitigate his damages can be taken into account in determining the amount of the plaintiff's damages.

Question 1310 A newspaper published an editorial in which an editor asserted that a candidate for high political office was a user of illegal drugs. The accusation was untrue. The editor acted unreasonably in not investigating the accusation before publishing it; however, the editor honestly believed that the accusation was true. The candidate sued the editor for defamation. Is the candidate entitled to recover? Answers: No, because the accusation appeared in an editorial and was, therefore, merely an opinion. No, because the editor honestly believed that the accusation was true. Yes, because calling someone an illegal drug user is defamatory per se. Yes, because the accusation was false and was injurious to the candidate's reputation.

Answer choice B is correct. In a defamation action brought by a candidate for public office, the plaintiff must establish more than mere negligence with regard to the truth or falsity of the allegedly defamatory statement of fact. The plaintiff must establish that the defendant acted with actual malice, that is, that the defendant in fact knew the statement to be false or acted with reckless disregard as to the truth of the statement. Here, the candidate cannot establish actual malice on the part of the editor in publishing the statement because the editor did not entertain serious doubts about its truthfulness. Answer choice A is incorrect. The assertion that the candidate used illegal drugs purported to be a statement of fact, not a statement of opinion. Defamation turns on what is conveyed in the statement published by the defendant. The context may influence what is conveyed, but facts may be stated in editorials or advertisements as well as in news reports. In this case, it is true that the candidate will not recover, but the reason is that he cannot show the actual malice required to defame a political candidate because the editor believed the statement about the candidate's drug use to be true. Acting unreasonably in not investigating the accusation does not necessarily rise to a reckless disregard for its truth. Answer choice C is incorrect. The plaintiff in a defamation action must establish that a statement is defamatory, and accusing someone of a criminal act is indeed "defamatory per se." However, a political candidate, like a political official, must also establish that the defendant acted with actual malice. Here, the candidate cannot establish that essential element of his case. Answer choice D is incorrect because the evidence does not support a finding of actual malice in this case.

Question 1376 In the parking lot of a mall, two drivers, while simultaneously backing up, each carelessly struck the other's car. Neither suffered physical injuries, but the damages to the plaintiff's car totaled $10,000; damages to the defendant's car were $1,000. The plaintiff sued to recover the damages to his car and the defendant counterclaimed for the damages to her car. At trial, it was determined that the plaintiff was 70% at fault and the defendant 30% at fault for the accident. The applicable jurisdiction has enacted a pure comparative negligence statute. After taking into account his own liability to the defendant, how much may the plaintiff recover from the defendant? Answers: Nothing $2,300 $3,000 $9,000

Answer choice B is correct. In a pure comparative negligence jurisdiction, the plaintiff is not precluded from recovering if his fault exceeds the defendant's fault. Because the defendant was 30% at fault, she is liable for 30% of the plaintiff's damages (i.e., $3,000). Because the plaintiff was 70% at fault, he is liable for 70% of the defendant's damages (i.e., $700). Offsetting the defendant's recovery from the plaintiff's recovery, the plaintiff is entitled to recover $2,300 (i.e., $3,000 - $700). Based on this analysis, answer choices A, C, and D are incorrect.

Question 7204 During a family cookout, a homeowner's new deck collapsed from the weight of the guests and the deck furniture. Many people, including the homeowner, suffered physical injuries as a result. The homeowner has sued the contractor who built the deck for negligence. At trial, it was established that the contractor used clearly rotted wood and rusted brackets that he salvaged from a junk yard to construct the deck, and that he did not properly tighten many of the fastening elements of the deck. During her presentation of evidence, the homeowner also entered local building safety codes into evidence. These safety codes recommended the use of new materials to build weight-bearing structures, such as decks, but imposed no penalty for failure to do so. In order to recover damages, is the homeowner required to present expert testimony regarding the standard of care to which the contractor should be held? Answers: No, because the homeowner has already established the applicable standard of care by offering the safety codes into evidence. No, because the jury can objectively determine what a reasonably prudent person would do under the circumstances without expert testimony. No, because the jury can rely on the judge to advise whether there was a breach. Yes, because expert testimony is required to establish a standard of care when the defendant is a professional in his field.

Answer choice B is correct. In most cases, the standard of care imposed is that of a reasonably prudent person under the circumstances. This standard is an objective one, measured by what a reasonably prudent person would do, rather than whether a particular defendant is acting in good faith or using her best efforts. In most situations, no expert testimony is required to define this standard of care, particularly when the defendant's negligence is so apparent that a layperson can identify it without the assistance of an expert in the field. Therefore, the jury is able to determine the standard of care applicable in this situation without expert testimony because the contractor used rusty brackets, rotting wood, and did not properly tighten the fastening elements on the deck. Answer choice A is incorrect. In some cases, a statute can be conclusive evidence of a standard of care. A violation of these statutes is considered negligence per se. However, the criminal or regulatory statute must impose a penalty for a violation of a duty it creates for this concept to apply. Because these safety codes impose no penalty, they cannot be used to conclusively establish a standard of care. Instead, they are merely evidence of negligence to be considered in the jury's objective determination of the proper standard of care in this case. Answer choice C is incorrect because the judge is regarded as having no more knowledge than the average lay person. If the claim were beyond the understanding of the average lay person, expert testimony would be necessary. Furthermore, it is the responsibility of the trier of fact—in this case, the jury—to determine whether there was a breach; for the judge to decide the issue would be improper. Answer choice D is incorrect because it misstates the law. Expert testimony is not always required to establish a standard of care for a professional. Although it may be helpful when it is necessary to explain what standard of care an ordinary practitioner in the same community would use, expert testimony is not required when the defendant's negligence is so apparent that a layperson could identify it.

Question 1419 A driver was traveling in his car one evening when he heard his cell phone ring. The phone was located on the console of the car, and the driver reached out to grab it. While the driver was looking at his caller identification display, a pedestrian walked into the crosswalk straight ahead of the vehicle. When the driver saw the pedestrian in the crosswalk, he swerved to the right to avoid the collision; in doing so, he slammed his car into a utility pole. The utility pole crashed to the ground. A power line attached to the utility pole snapped in two, resulting in power outage within the surrounding area. One block away from the accident site, a child was watching television when the power went out, and her house became pitch black. The child, who was scared of the dark, ran out of the house and into the street where the child was struck by a bicyclist and seriously injured. The child's parents sued the driver of the car that crashed into the utility pole, claiming that the child's injury was attributable to the driver's negligence. Who should prevail? Answers: The driver, because his actions were not the cause in fact of the child's injuries. The driver, because he owed no duty to the child. The parents, because the driver's conduct was the legal and proximate cause of the child's injuries. The parents, because the driver owed a duty of reasonable care to the child.

Answer choice B is correct. The majority of jurisdictions follow the "Cardozo view" of duty. Under this view, a defendant is liable only to a plaintiff who is within the "zone of foreseeable harm." Here, the child was one block from the accident site, and her injuries occurred in a way that was unforeseeable at the time of the accident. Thus, she is an "unforeseeable plaintiff," and the driver owed her no duty of reasonable care. Answer choice A is incorrect because but for the driver's actions, the child would not have been injured. Answer choice C is incorrect because even if it were true (which it very likely is not), it ignores the issue of duty, which must be addressed before causation. Answer choice D is incorrect because it articulates the "Andrews view," which is followed only by a minority of jurisdictions. That view posits that, if if the defendant can foresee harm to anyone as a result of his negligence, then a duty is owed to everyone (foreseeable or not) harmed as a result of his breach.

Question 7206 A landowner became concerned when a large apple tree on the edge of her property grew so large that it drooped over her privacy fence and dropped apples onto the public sidewalk. The landowner, having no personal experience with landscaping and knowing that removing a tree of this size was inherently dangerous, hired a landscaper to remove the tree. However, the landscaper negligently failed to attach guidelines to the tree before he cut it down. Due to this oversight, when the landscaper chopped the trunk of the tree, it toppled through the fence, striking a pedestrian who was walking on the public sidewalk on the other side of the fence. The pedestrian sued both the landscaper and the landowner for his injuries. What is the pedestrian's strongest argument to recover damages from the landowner? Answers: The landowner assumed liability for the landscaper's conduct by paying him. The landowner is liable because she hired the landscaper to perform an inherently dangerous service. The landowner is liable for the landscaper's negligence under the theory of respondeat superior. The landowner is strictly liable for damages caused by a naturally dangerous condition on her property.

Answer choice B is correct. Those who engage an independent contractor are generally not vicariously liable for the torts of the independent contractor. However, a person who hires an independent contractor remains vicariously liable for inherently dangerous activities. Therefore, the pedestrian's best chance for recovering damages from the landowner is to argue that tree removal is an inherently dangerous service, and so the landowner is still liable for damages caused by the landscaper's performance. Answer choice A is incorrect because someone who hires an independent contractor to perform a service is not automatically liable for the independent contractor's negligence simply because he pays the independent contractor. Instead, the service must relate to a non-delegable duty, such as an inherently dangerous activity. Answer choice C is incorrect because the theory of respondeat superior applies to an employer's vicarious liability for the torts of an employee. However, this landscaper is more likely an independent contractor than an employee. An independent contractor is one engaged to accomplish a task or achieve a result but who is not subject to another's right to control the method and means by which the task is performed or the result reached. They tend to have specialized skills or knowledge, and tend to work for many employers. Here, the landscaper fits that description, and respondeat superior is unlikely to apply. Answer choice D is incorrect because strict liability will not apply in this situation.

Question 1330 A manufacturer sells shoes with the manufacturer's brand name to two competing retailers. When the larger of the two retailers, a national retailer, learns that the manufacturer is also selling the same brand of shoes to its competitor, the retailer stops placing orders for the shoes with the manufacturer. In response to a query by the manufacturer as to the reason for larger retailer's action, the large retailer states that it does not want to carry the same brand name shoes as the smaller regional retailer. Under no contractual obligation to sell the brand name shoes to the regional retailer, the manufacturer stops doing so. The national retailer resumes ordering the brand name shoes from the manufacturer. The regional retailer, while suffering a loss of income due to its inability to sell the brand name shoes, continued to be profitable since shoe sales represented only a small fraction of its overall sales. In an action for intentional interference with a contract brought by the regional retailer against the national retailer, which of the following would be the national retailer's weakest argument? Answers: There was no contract between the manufacturer and the regional retailer. The national retailer did not substantially interfere with the regional retailer's overall business. The regional retailer was a competitor. The national retailer is merely exercising its freedom to refuse to deal with another.

Answer choice B is correct. To prove intentional interference with a contract, the regional retailer must prove that (1) a valid contract existed between the regional retailer and the manufacturer, (2) the national retailer knew of the contractual relationship, (3) the national retailer intentionally interfered with the contract, causing a breach, and (4) the breach caused damages to the the regional retailer. The regional retailer need only show a pecuniary loss of the benefits from the contract; there is no requirement that the interference prevent the regional retailer from being profitable or that the loss be substantial. Therefore, the argument that the national retailer did not substantially interfere is a weak argument, and answer choice B is correct. Answer choice A is incorrect because when the interference involves a prospective economic relationship, rather than an existing contractual relationship, the defendant has greater latitude in acting in its own interest, particularly if the plaintiff is a competitor. Therefore, answer choice A would be a strong argument for the national retailer to make. Answer choice C is also a strong argument and therefore incorrect because the relationship between the parties is a factor in determining whether the interference is improper. Generally, a competitor has greater latitude in acting in a manner that interferes with the business relationships of a competitor. Answer choice D is incorrect because a person is generally free to deal or to refuse to deal with another. Thus, this argument is stronger than answer choice B.

Question 6174 An acrobat went to a chiropractor when he pinched a nerve in his neck while practicing an acrobatic routine. The chiropractor had the acrobat lay down on a custom-made chiropractic table. The table could be tilted such that the acrobat was angled with his head towards the floor, allowing the chiropractor deeper access to the acrobat's neck muscles. During the session, the table malfunctioned due to a manufacturing defect and tilted all the way down, causing the acrobat to bang his head on the floor. As the acrobat tried to pull himself up, he frantically reached out and grabbed the chiropractor's shoulder, wrenching it and causing serious injury. If the chiropractor brings a strict liability action against the manufacturer of the table to recover damages for his shoulder injury, who will prevail? Answers: The chiropractor, because he was within the zone of danger of a threatened physical impact. The chiropractor, because the chiropractic table was used for its intended purpose. The manufacturer, because the chiropractor is not the proper plaintiff for this action. The manufacturer, because the chiropractor's shoulder injury did not occur in a reasonably foreseeable way.

Answer choice B is correct. To recover under a strict products liability theory, the plaintiff must plead and prove that the product was defective, the defect existed when it left the defendant's control, and the defect caused the plaintiff's injuries when the product was used in an intended or reasonably foreseeable way. Here, the facts state that the table malfunctioned due to a manufacturing defect. Thus, it can be presumed that the defect existed when it left the manufacturer's control. The table was also being used for its intended purpose when the injury occurred. Therefore, the chiropractor will prevail under a strict products liability theory. Answer choice A is incorrect because it refers to the legal standard for negligent infliction of emotional distress. Answer choice C is incorrect because anyone foreseeably injured by a defective product or whose property is harmed by the product may bring a strict-liability action. Appropriate plaintiffs include purchasers, other users of the product, and even bystanders who suffer personal injury or property damage. Answer choice D is incorrect because it is the product that must be used in an intended or reasonably foreseeable way. Moreover, it was likely reasonably foreseeable in this situation that the chiropractor would be injured in such a way.

Question 7217 A rancher kept his cows in a pasture beside a busy road. One evening, a cow knocked down a short portion of the fence separating the pasture from the road and wandered into the street. A motorist driving down the road turned the corner and saw the cow. Even though the motorist was driving at a reasonable speed, he knew he could not stop before he hit the cow. Knowing that hitting an animal that size could cause a fatal car accident, the motorist veered off of the road and into the rancher's pasture, damaging a long stretch of the rancher's fence in the process. If the rancher sues the motorist for trespass to land to recover damages, is he likely to succeed? Answers: Yes, because the motorist voluntarily entered the rancher's property. Yes, because the motorist caused actual damages to the rancher's fence. No, because the motorist can claim the privilege of private necessity. No, because the rancher was responsible for keeping his cows in the pasture.

Answer choice B is correct. Trespass to land occurs when the defendant's intentional act causes a physical invasion of the land of another. Private necessity is a qualified privilege to protect an interest of the defendant or a limited number of other persons from serious harm. The privilege applies if the interference was reasonably necessary to prevent a serious injury from nature or another force not connected with the property owner. Despite this privilege, the property owner is entitled to recover actual damages, but cannot recover nominal or punitive damages nor use force to eject the defendant. Here, the privilege of private necessity applies, but the rancher will still be able to recover for the actual damage done to his fence. Answer choice A is incorrect because this fact is insufficient to establish that the rancher can recover damages. If the motorist had entered the property with the privilege of private necessity and caused no damages, the rancher would not have been permitted to recover damages. Answer choice C is incorrect because the privilege of private necessity does not prevent the property owner from collecting actual damages. Answer choice D is incorrect. Because the defendant need only enter the land with the intent to do so, the rancher's own negligence does not affect the motorist's liability for this intentional tort. However, the general negligence standard applies if an animal strays onto a public road and contributes to an accident there, so the motorist would likely raise this negligence in a counterclaim, possibly negating his having to actually pay the rancher any damages.

Question 6555 An environmentalist purchased an alternative fuel vehicle from an automobile manufacturer who specialized in eco-friendly cars. The alternative fuel vehicle's engine was based upon a diesel engine, but was designed to run on vegetable oil instead of diesel fuel. Per the detailed instructions given by the manufacturer, the vegetable oil needed to be heated, filtered, and then mixed with a blend of chemical additives in order for it to be safely used in the vehicle's engine. An engine that would safely combust untreated vegetable oil was possible, but the cost was prohibitive to the manufacturer. Instead, the manufacturer provided the equipment, the blend of chemicals additives, and instructions necessary to convert the untreated vegetable oil into a usable form of fuel. The environmentalist read the instructions but did not use the chemical additives because he did not want to disturb the natural state of the vegetable oil. While driving his vehicle for the first time, the engine started smoking and then caught on fire. The environmentalist sued the manufacturer for damages under a theory of strict liability based on a design defect. Is the environmentalist likely to succeed in a jurisdiction that applies the risk-utility test? Answers: No, because the environmentalist's failure to use the chemical additives will bar his action. No, because the cost of a reasonable alternative engine design was prohibitive to the manufacturer. Yes, because the design of the vehicle's engine rendered the vehicle not reasonably safe. Yes, because the foreseeable risk of the engine catching on fire was not obvious to the environmentalist.

Answer choice B is correct. Using the risk-utility test, to prevail on a claim under a strict-products-liability-design-defect theory, the jury must determine whether the risks posed by the product outweigh its benefits. A plaintiff generally must prove that a reasonable alternative design was available to the defendant and the failure to use that design has rendered the product not reasonably safe. The alternative design must be economically feasible. Here, the manufacturer could have made an engine that could combust untreated vegetable oil, but it would not have been economically feasible. Accordingly, the environmentalist's claim will not succeed. Answer choice A is incorrect because foreseeable misuse, alteration, or modification usually does not preclude recovery. Here, the environmentalist's misuse of the vehicle's engine by failing to use the chemical additives was foreseeable enough to warrant a manufacturer's warning and detailed instructions. Therefore, this form of misuse was likely foreseeable, and although it may have reduced any damage award, it is unlikely to bar the environmentalist's recovery. Answer choice C is incorrect. The design of the vehicle's engine likely did not render the vehicle not reasonably safe because the engine could be operated safely, had the user followed the instructions provided by the manufacturer. Answer choice D is incorrect because it states the test for a failure-to-warn theory of recovery, and is inapplicable to this action.

Question 1368 In what began as consensual roughhousing on a public sidewalk, a high school freshman refused to stop despite a request by the other, much larger participant, a college senior. As a consequence, the senior began to pummel the freshman. The freshman managed to escape from the senior, and fled screaming for help with the senior in pursuit. A deliveryman, who was unacquainted with either student, witnessed only the freshman's flight from the senior. The deliveryman, responding to the freshman's appeal, used reasonable force to trip the senior as he came running by the deliveryman. As a result, the senior suffered physical injuries. The senior brought suit against the deliveryman to recover damages for his injuries. Who will prevail? Answers: The senior, because the deliveryman purposefully caused the senior's injuries. The senior, because the deliveryman was not related to the freshman. The deliveryman, because he acted in defense of the freshman. The deliveryman, because the senior consented to the roughhousing.

Answer choice C is correct because one is justified in using reasonable force in defense of others upon reasonable belief that the defended party would be entitled to use self-defense. Here, the deliveryman acted to protect the freshman from an assault by the senior and used reasonable force to do so. Although the freshman became the aggressor by refusing to stop roughhousing despite the senior's request, the senior, by chasing the freshman, was acting in retaliation rather than in self-defense. Answer choice A is incorrect because, although the deliveryman did intentionally cause the senior's physical injuries, the deliveryman was acting in defense of the freshman. Answer choice B is incorrect because the defense of others is not limited to actions to protect a family member. Answer choice D is incorrect because, although the senior did initially consent to the roughhousing, that consent was vitiated by the senior's request to terminate the roughhousing.

Question 6108 An elderly woman living alone in a city was diagnosed with a heart condition. On hearing the news, her grandson told her he wished he could move closer to help her with housework and errands, but that he could not afford the rent. To help her grandson afford to move into her apartment complex, the elderly woman offered to cosign the grandson's lease as a surety. The grandson agreed and moved into the neighboring apartment. Even before his first rental payment was due, the grandson started asking the elderly woman for money to cover his rent and living expenses. When she refused to give him money directly, he began to bang on the adjoining wall of their apartments at all hours of the night, yelling and threatening to move away and leave her to fend for herself if she did not pay his rent. His threats terrified the woman, and her health deteriorated from the stress and lack of sleep. One night when the grandson banged on the wall and yelled that he would stop picking up her prescriptions unless she gave him money, the elderly woman had a heart attack. If the elderly woman files a claim against the grandson for intentional infliction of emotional distress, under the majority rule, is she likely to prevail? Answers: No, because liability for emotional distress does not extend to mere abusive language. No, because the elderly woman had a preexisting hypersensitivity. Yes, because the grandson knew about the elderly woman's age and heart condition. Yes, because the woman's heart attack satisfies the physical injury requirement.

Answer choice C is correct. A defendant is liable for intentionally or recklessly acting with extreme and outrageous conduct that causes the plaintiff severe emotional distress. Conduct is extreme and outrageous if it exceeds the possible limits of human decency, so as to be entirely intolerable in a civilized society. Courts are more likely to find a defendant's abusive language and conduct to be extreme and outrageous if the plaintiff is a member of a group with a known heightened sensitivity (e.g., an elderly woman). If the plaintiff is hypersensitive and experiences severe emotional distress unreasonably, then there is no liability unless the defendant knew of the plaintiff's heightened sensitivity. Here, the elderly woman is a member of a group with a known heightened sensitivity because she is elderly, so the court is more likely to find the grandson's threats and repeated harassment to be extreme and outrageous. He engaged in this conduct either intending to cause such severe emotional distress that the woman would relent and give him money, or with recklessness as to the risk of causing such distress. Her heart condition likely qualifies as a hypersensitivity, but because the grandson knew about her condition, this will not be a defense to liability for the elderly woman's heart attack. Answer choice A is incorrect because a defendant's abusive language and conduct may be sufficiently "extreme and outrageous" if either (i) the defendant is in a position of authority or influence over the plaintiff, or (ii) the plaintiff is a member of a group with a known heightened sensitivity (e.g., elderly persons). Answer choice B is incorrect. In terms of damages, the plaintiff must prove severe emotional distress beyond what a reasonable person could endure. If the plaintiff is hypersensitive, however, and experiences severe emotional distress unreasonably, then there is no liability unless the defendant knew of the plaintiff's heightened sensitivity. Here, the grandson knew that the elderly woman was hypersensitive because he knew about her heart condition, so he is liable. Answer choice D is incorrect because most courts do not require physical injury.

Question 7207 A homeowner hired a landscaper to move a number of large boulders from the bottom of a hill on her property to the Zen garden she had built at the top of the hill. Unknown to the landscaper, the homeowner was a regular practitioner of fire poi, a unique performance art that involved swinging burning weights on long chains in a complex rhythmic and geometric choreography. The night after the landscaper placed the boulders, it rained heavily on the homeowner's property. The rain eroded some of the sand at the base of each boulder, leaving them unstable. The next morning, the homeowner went outside to practice poi while standing on one of the boulders. She lost her balance when an unstable boulder shifted, and she suffered severe burns. The homeowner sued the landscaper for negligence, and established at trial that the rain would not have affected the stability of the boulders if the landscaper had placed them slightly deeper into the sand. She has also established that she was very proficient at fire poi, and would not have burned herself if the boulder had been steady. On these facts, which of the following is the landscaper's strongest defense? Answers: The homeowner assumed a risk of injury by practicing poi on a boulder. The homeowner was engaged in an abnormally dangerous activity. The landscaper could not reasonably foresee the homeowner's injuries. The rain was a superseding cause of the homeowner's injuries.

Answer choice C is correct. A defendant is liable for the reasonably foreseeable consequences resulting from his conduct. The type of harm must be foreseeable, and a defendant's liability is limited to those harms that result from the risks that made the defendant's conduct tortious. The homeowner is suing the landscaper for the negligent placement of the boulders. Foreseeable consequences of such negligence may have included damage caused if the boulders rolled, of even damages sustained in a fall from the boulder. However, burns are not a foreseeable type of harm within the scope of the risk created by poor placement of the boulders. Therefore, the landscaper should argue that he is not liable for these unforeseeable injuries. Answer choice A is incorrect. Traditionally, a plaintiff's voluntarily encountering a known, specific risk is an affirmative defense to negligence that affects recovery. Most courts hold that the voluntary encountering must also be unreasonable. Here, given the level of proficiency the homeowner has established, it is unlikely that her practice was unreasonable, or that she carried out her practice in the face of a known risk that the boulder would shift. Additionally, in most comparative-fault jurisdictions, this form of assumption of the risk merely reduces recovery. Therefore, answer choice C offers a stronger defense. Answer choice B is incorrect because, even if fire poi is an abnormally dangerous activity, that alone does not bar the homeowner from recovering damages. Instead, it may have made her strictly liable to bystanders, and it may qualify as contributory negligence. However, in pure comparative negligence jurisdictions (which is the default on the MBE), her negligence would not bar her recovery. Answer choice D is incorrect. Examples of unforeseeable superseding causes include extraordinary acts of nature ("act of God"), criminal acts, and intentional torts of third parties. However, normal forces of nature, including rain, are considered foreseeable intervening causes, and will not limit the defendant's liability.

Question 6649 A professional football player was severely injured during a football game. During the game, a player from the opposing team crushed the football player's jaw when he ripped off his football helmet and intentionally kneed him in the head in order get possession of the football. The plaintiff-football player brought a battery action seeking compensation from the opposing player. During the trial, the defendant-football player introduced evidence demonstrating that all of the players, including the plaintiff, use rough tactics when tackling players on the opposing team in order to obtain possession of the ball. The plaintiff-football player introduced evidence that intentionally hitting a player once his helmet had been removed was beyond the scope of the game. Will the plaintiff-football player likely prevail in an action for damages against the defendant? Answers: No, because having used rough tactics during the football game, he was estopped from suing. No, because the defendant did not intend to cause a harmful or offensive contact to the plaintiff. Yes, because the defendant's reckless disregard for the plaintiff's safety was not consented to. Yes, because the defendant intentionally pulled off the plaintiff's helmet.

Answer choice C is correct. A defendant is liable to the plaintiff for battery when he causes a harmful or offensive contact with another person, and acts with the intent to cause such contact or the apprehension of such contact—unless the person consents. Consent may be implied by custom or usage—for example, participation in a contact sport. However, a participant in an athletic contest can recover if the defendant shows reckless disregard for a player's safety, such as a violation of a safety rule designed primarily to protect participants from serious injury. Here, the defendant showed reckless disregard when he ripped the helmet off of plaintiff's head and then purposely hit the plaintiff in the head after the fact. A football helmet is designed primarily to protect football players from serious head injuries caused by tackling and rough plays. Because the plaintiff introduced evidence that this behavior is beyond the scope of the game, and therefore beyond the scope of consent, the defendant may be liable for battery. Answer choice A is incorrect because the defendant exceeded plaintiff's implied consent. Using "rough" tactics is not the same as using tactics that are beyond the scope of the game and the scope of consent. Therefore, C is a better answer. Answer choice B is incorrect because defendant did intend to cause a harmful or offensive contact when he violently ripped the helmet from plaintiff's head and then kneed him. Answer choice D is incorrect because although intent is an essential element of battery, the plaintiff-football player's implied consent to some rough tactics negates the battery. Thus, the better answer is choice C because it focuses on the central issue—that the defendant's use of force exceeded the consent because of the defendant's reckless disregard for the plaintiff-football player's safety.

Question 7215 An elderly woman moved from her home in a rural community into an assisted living facility in the city. The woman was partially deaf, but was otherwise in good health. She walked from the living facility to the grocery store every day, despite her irrational but honestly held belief that she would be mugged. To protect herself, the woman kept pepper spray in her purse on her walks. One day, when the woman was walking back to the facility with her groceries, the bag tore and her produce rolled all over the sidewalk. A young man walking on the other side of the street saw what happened and called out "Hold on miss, I'll help!" The woman did not hear what he said, but did see him look both ways and then jog across the busy street. The man slowed and smiled as he approached her. Honestly believing she was being attacked, the woman pepper sprayed the man in the face. The man has sued the woman for battery, and the woman has asserted that she acted with the privilege of self-defense. If the man succeeds in his action, what is the most likely reason? Answers: There is no evidence that the woman attempted to retreat before using force. The woman's partial deafness was not taken into account in her claim of self-defense. The woman's belief, while honestly held, was not reasonable. As the initial aggressor, the woman is not entitled to claim self-defense unless the other party responded with deadly force.

Answer choice C is correct. A person may use reasonable force to defend against an offensive contact or bodily harm that he reasonably believes is about to be intentionally inflicted upon him. A person's mistaken belief that he is in danger, so long as it is a reasonable mistake, does not invalidate the defense. Under the circumstances, it is probably not reasonable to assume that a man who jogged across a busy street and smiled at the woman after she had dropped her groceries planned to attack her. Answer choice A is incorrect because the majority of jurisdictions do not require an attempt to retreat before the use of force. Answer choice B is incorrect. A person's particular physical characteristics, such as blindness or deafness, are taken into account when determining the reasonableness of their actions. However, even taking the elderly woman's deafness into account here does not make her reaction reasonable; further, even a person on average hearing ability may not have been able to hear the man across the busy street. Therefore, even considering this physical characteristic, the woman's belief was not reasonable. Answer choice D is incorrect. When a defendant was the initial aggressor because she was responding to an honest and reasonable belief that an offensive contact or bodily harm was about to be inflicted upon her, she could claim self-defense. Therefore, even though the elderly woman attacked the man first, she could have claimed self-defense if her belief was reasonable. There is no requirement of a threat of deadly force.

Question 7105 A kayaker on a lake did not notice when a boat towing a water skier approached him from behind. As the boat passed the kayaker, the boat driver thought it would be funny to swerve quickly around the kayak, causing a violent wake in that area of the lake that he hoped would scare the water skier by knocking him into the water. The boat driver quickly turned in front of the kayak, alarming the kayaker and causing a wave that almost flipped the kayak. In attempting to keep his balance, the water skier behind the boat involuntarily struck the back corner of the kayak. The impact, combined with the wake, flipped the kayak. The boat driver saw the incident but did not stop. The kayaker was terrified when his kayak flipped over, and he was trapped under the kayak for a few moments before he was able to free himself. He subsequently had to see a therapist to get over his new fear of water. If the kayaker files suit against the boat driver, for which of the following might the kayaker recover? Answers: Only negligent infliction of emotional distress Only intentional infliction of emotional distress Battery and negligent infliction of emotional distress Battery and intentional infliction of emotional distress

Answer choice C is correct. A plaintiff can recover for negligent infliction of emotional distress from a defendant if the plaintiff demonstrates that he was within the "zone of danger" of a threatened physical impact, that he feared for his own safety because of the defendant's negligence, and the threat of physical impact caused emotional distress. Here, the kayaker was within the zone of danger of the threatened impact of the wake intentionally caused by the boat driver's erratic driving. This threat of physical impact also caused the kayaker emotional distress for which he had to see a therapist. The extreme fear of water is likely a sufficient physical manifestation of harm, similar to nightmares, to satisfy such a requirement in most jurisdictions that require it. These facts satisfy the elements of a claim for negligent infliction of emotional distress. The boat driver is also liable for battery. A defendant is liable to a plaintiff for battery when he causes a harmful or offensive contact with the person of another, and acts with the intent to cause such contact or the apprehension of such contact. Here, the boat driver intended a harmful and offensive contact against the water skier when he tried to make him fall into the water. However, under the doctrine of transferred intent, the boat driver is also liable to the kayaker for battery because he intended to commit a battery against the water skier and inadvertently caused a harmful contact to the kayaker. Answer choice A is incorrect because the doctrine of transferred intent also makes the boat driver liable for battery. Answer choices B and D are incorrect. A defendant may be liable for intentionally or recklessly acting with extreme and outrageous conduct that causes the plaintiff severe emotional distress. Here, the driver did not intentionally try to turn the kayaker's kayak over, and his actions are probably merely negligent rather than extreme and outrageous.

Question 7211 A large oil company purchased a plot of land in a rural area and erected an oil well in compliance with local zoning codes. Many families living in the surrounding acres could hear the oil well operating loudly at all hours of the day and night. There was also a small for-profit botanical garden nearby. For years, the garden had drawn most of its customers with a bird-watching hike during the migration period of a rare type of songbird. The persistent sounds of operations at the oil well drove the songbirds to avoid the botanical garden during their migration. The noise from the oil well and the lack of songbirds caused the garden to lose most of its visitors, and it faced bankruptcy as a result. The garden brought an action based on nuisance against the oil company for its economic loss. Is the garden likely to succeed in its action? Answers: No, because the oil well was in compliance with the local zoning codes. No, because the sounds of operations at the oil well constitute a public nuisance. Yes, because the garden has suffered a substantial economic loss. Yes, because the oil company installed the oil well after the garden had established a business.

Answer choice C is correct. A public nuisance is an unreasonable interference with a right common to the general public. A private citizen has a claim for public nuisance only if she suffers harm that is different in kind from that suffered by members of the general public. Here, the incessant noise from the oil well operations has driven the songbirds away from the area. This has had a significant economic effect on the garden, which is a harm different in kind from that suffered by the general public. Accordingly, the garden will succeed in its action. Answer choice A is incorrect. The fact that a defendant complies with a statute, local ordinance, or administrative regulation is not a complete defense to a nuisance action. It is only one piece of evidence to be considered in determining whether an interference is unreasonable. Answer choice B is incorrect. The incessant noise from the oil well operations likely constitutes a public nuisance because a number of families living in the surrounding acres could hear the oil well operating loudly at all hours of the day and night. Even though the operations constitute a public nuisance, the garden can bring a nuisance claim because it is suffering a particularized economic loss as a result of the noise. Answer choice D is incorrect because the fact that the garden did not "come to" this nuisance is not determinative of its success in this action.

Question 6001 An on-duty firefighter was injured when he fell down as a consequence of poorly-maintained stairs in the common area of an apartment building. The maintenance of the stairs was the responsibility of the owner of the building. The fire was caused by the negligence of an inebriated tenant who had fallen asleep while smoking a cigarette in bed. Which of the following best describes the likely outcome of the firefighter's tort claim? Answers: The firefighter can recover from either the owner or the tenant. The firefighter can recover from the tenant, but not the owner. The firefighter can recover from the owner, but not the tenant. The firefighter cannot recover from either the owner or the tenant.

Answer choice C is correct. An emergency professional, such as a police officer or firefighter, is barred from recovering damages from the party whose negligence caused the professional's injury if the injury results from a risk inherent in the job ("firefighter's rule"). In this case, the firefighter's injury was caused by the owner's failure to properly maintain the stairs leading into the apartment building. This injury did not result from a risk inherent in the job and consequently that firefighter can recover from the owner. The firefighter's rule would, however prevent the firefighter from recovering from the tenant whose negligence caused the fire which, in turn, led to the firefighter's presence in the apartment building where the firefighter was injured. Answer choice A is incorrect because the firefighter can only recover from the owner. Answer choice B is incorrect because the firefighter cannot recover from the tenant. Answer choice D is incorrect because the firefighter can recover from the owner, but not the tenant.

Question 1299 A mining company that operated a copper mine in a remote location kept dynamite in a storage facility at the mine. The storage facility was designed and operated in conformity with state-of-the-art safety standards. In the jurisdiction, the storage of dynamite is deemed an abnormally dangerous activity. Dynamite that was stored in the mining company's storage facility and that had been manufactured by an explosives manufacturer exploded due to an unknown cause. The explosion injured a state employee who was at the mine performing a safety audit. The employee brought an action in strict liability against the mining company. What would be the mining company's best defense? Answers: The mine was in a remote location. The mining company did not manufacture the dynamite. The state employee assumed the risk of injury inherent in the job. The storage facility conformed to state-of-the-art safety standards.

Answer choice C is correct. Assumption of risk can be an affirmative defense to strict liability, and in this case, the state employee willingly took on auditing duties in potentially dangerous environments. Answer choice A is incorrect. The location of the activity and whether the activity is common for the area are factors that might be considered in determining whether an activity is "abnormally dangerous," but the facts specify that the state has already determined that the activity of storing dynamite is abnormally dangerous without reference to the location at which it is conducted. Moreover, the storage of dynamite even in remote locations is often considered to be abnormally dangerous. Answer choice B is incorrect. In a products liability action, the defendant's status as a manufacturer or other "seller" of the product would be important, but this strict liability action is based on an abnormally dangerous activity rather than on a defective product. In a strict liability action based on engaging in an abnormally dangerous activity, a defendant can be held liable for all the risks inherent in the activity, whatever other actors might be involved in creating those risks. Answer choice D is incorrect. The storage facility's conformity to the highest safety standards would be relevant if the action were based on negligence. But in an action based on strict liability for engaging in an abnormally dangerous activity, the defendant will be held responsible even for those risks that could not have been avoided through the use of extraordinary care.

Question 6533 A court clerk took a quick afternoon break to grab a cup of coffee from a stall located outside of the courthouse building. As he was about to exit the building, he walked past an elevator and noticed that a woman's foot was stuck in the elevator doors. As he walked by in a rush to get to the coffee stall, he heard the woman begging for him to get help because the elevator doors were stuck and she was in pain. Despite the woman's pleas, the clerk continued on his way to the coffee stall and did not assist the woman. The woman has filed a negligence claim against the clerk for the injuries she sustained to her leg resulting from her foot being stuck in the elevator doors for an extended period of time. Which of the following statements most accurately describes the clerk's liability? Answers: The clerk is liable because he assumed a duty to act with reasonable care. The clerk is liable because his failure to get help for the woman was unreasonable. The clerk is not liable because he did not have a duty to act. The clerk is not liable because he does not have a unique relationship with the woman.

Answer choice C is correct. In general, there is no affirmative duty to act, subject to certain exceptions that are not applicable in this case. Thus, the clerk is not liable. Answer choice A is incorrect. A person who voluntarily aids or rescues another has a duty to act with reasonable ordinary care in the performance of that aid or rescue. Here, the clerk did not voluntarily aid the woman or attempt to rescue her—he walked by her without doing anything. Because he did not assume the duty to help her, he is not required to act with reasonable ordinary care. Answer choice B is incorrect. Generally, there is no duty to act affirmatively, even if the failure to act appears to be unreasonable. Answer choice D is incorrect. The mere fact that the clerk does not have a unique relationship with the woman does not prohibit the clerk's liability for the woman's injuries. The clerk would still be liable if he assumed a duty to rescue the woman, placed the woman in peril, or had a contractual obligation, even if he did not have a unique relationship with the woman. Because the clerk did not have an affirmative duty to act here on any basis, he is not liable.

Question 4302 Immediately before leaving a party, a college student ate a cookie that, unknown to her, was laced with a hallucinogen. On the drive home, she lost control of her car as a consequence of the hallucinogen and struck another vehicle. The driver of the other vehicle sued the student under a negligence theory to recover for damages caused by the accident. At trial, the judge instructed the jury that, under the reasonable person standard, the student's driving was to be judged in light of the care that a sober person in her situation would have exercised. Has the judge correctly instructed the jury? Answers: Yes, because a sober person is a reasonable person. Yes, because the student's physical characteristics cannot be considered in determining whether she violated the reasonable person standard. No, because the student unknowingly ingested the hallucinogen. No, because the student was intoxicated at the time of the accident.

Answer choice C is correct. In most cases, the standard of care imposed is that of a reasonably prudent person under the circumstances, as measured by an objective standard. Intoxicated individuals are held to the same standards as sober individuals unless their intoxication was involuntary. Here, the student's intoxication was involuntary, as she did not know the cookie contained any illicit substance. Therefore, she should not be held to the standard of a sober person. Answer choice A is incorrect because, while a sober person may be a reasonable person, that isn't necessarily the case. Moreover, an intoxicated individual is not held to the same standard as a sober individual when the intoxication is involuntary. Answer choice B is incorrect because intoxication is not considered a physical characteristic. Answer choice D is incorrect because being intoxicated does not necessarily relieve one of liability; the issue is whether the intoxication was voluntary or not.

Question 1348 The manufacturer of hot water heaters contacted the supplier of plastic resin about using the resin in the manufacturing of the heater. The supplier gave the manufacturer technical advice about how to mold the resin into a hot water tank. The supplier told the manufacturer that, in order to withstand the temperatures specified by the manufacturer, the resin would need to be at least one inch thick. The manufacturer ordered the resin from the supplier. The manufacturer designed and made the tank for its hot water heaters three-quarters of an inch thick using the supplier's resin. A plumber purchased a hot water heater made by the manufacturer from a local plumbing supply store and installed it in a homeowner's residence. Due solely to the walls of the tank being too thin, the tank melted when used by the homeowner. The homeowner suffered both severe burns as well as substantial property damage as a consequence of the melted tank. The homeowner initiated a strict products liability action against the plastic supplier and the local plumbing store for damages suffered as a consequence of the melted hot water tank. Who will be liable? Answers: Both the plastic supplier and the plumbing store The plastic supplier only The plumbing store only Neither the plastic supplier nor the plumbing store

Answer choice C is correct. The plumbing store is liable for the harm caused to the plaintiff by the defective product (i.e., a hot water heater). To be subject to strict liability for a defective product, the defendant must be in the business of selling or otherwise distributing products of the type that harmed the plaintiff. Included as a seller are the manufacturer of the product, its distributor, and its retail seller. As long as the seller is a commercial supplier of the product, the seller is subject to strict liability for a defective product even if the seller was not responsible for the defect in any way. Here, the product is defective due to its design; the walls of the tank were too thin. Although the local plumbing store did not manufacture the defective hot water heater, the store, as a commercial supplier of the product, is strictly liable for it. However, the supplier of the plastic is not liable. The commercial supplier of a component that is integrated into a product during its manufacture is not liable unless the component itself is defective or the supplier substantially participates in the integration process and the integration of the component causes the product to be defective. Here, the plastic resin provided by the supplier was not itself defective, and, although the supplier participated in the integration process, the supplier's participation did not lead to the defective hot water heater. Had the supplier's advice been followed, the hot water heater tank would not have melted. For the reasons enumerated for answer choice C, answer choices A, B, and D are incorrect.

Question 6004 A plaintiff was a very smart and successful high school student. He was also, unfortunately, the victim of some teasing by the other boys because he was physically weak and clumsy. In class one day, the defendant, one of the other boys, decided to pester the plaintiff by throwing a paper airplane at the plaintiff when the teacher was not looking. When the plaintiff tried to duck out of the way of the paper airplane, he awkwardly fell off his chair and hit his head on the desk, resulting in a concussion. The plaintiff brought a suit for battery against the defendant. Of the following, which would be the most important matter for the plaintiff to prove? Answers: That it was against school rules to throw paper airplanes in class. That the defendant intended to do him physical harm. That he did not consent to being pestered in this way. That his own weakness and clumsiness did not contribute to his injury.

Answer choice C is correct. There is no battery if the plaintiff consented to the act, either expressly or by virtue of participating in a particular event or situation. In this case, the facts do not indicate that the plaintiff expressly consented to the defendant's conduct. Similarly, the plaintiff did not consent to having objects thrown at him while in class merely by being present in the classroom. Answer choice A is incorrect because although the school rules would be relevant, they would be relevant only as evidence of whether the plaintiff did or did not implicitly consent. Answer choice B is incorrect because the tort of battery does not require a showing that the defendant intended to cause physical injury. Answer choice D is incorrect because the tort of battery entails liability for any harm even if the harm is worse than anticipated because of the weakness or clumsiness of the plaintiff.

Question 1292 A fire that started in the defendant's warehouse spread to the plaintiff's adjacent warehouse. The defendant did not intentionally start the fire, and the plaintiff can produce no evidence as to how the fire started. However, the defendant had failed to install a sprinkler system, which was required by a criminal statute. The plaintiff can produce evidence that had the sprinkler system been installed, it could have extinguished the fire before it spread. In an action by the plaintiff against the defendant to recover for the fire damage, is it possible for the plaintiff to prevail? Answers: No, because the statute provides only for criminal penalties. No, because there is no evidence that the defendant negligently caused the fire to start. Yes, because a landowner is strictly liable for harm to others caused by the spread of fire from his premises under the doctrine of Rylands v. Fletcher. Yes, because the plaintiff was harmed as a result of the defendant's violation of a statute that was meant to protect against this type of occurrence.

Answer choice D is correct. A criminal statute can be used to set the standard of care in a negligence action if it was meant to protect against the type of harm that occurred by specifying preventive steps that should be taken. In that case, violation of the statute is negligence per se. For these reasons, answer choice A is incorrect. Answer choice B is incorrect because even though there is no evidence that the defendant intentionally or negligently started the fire, his failure to take preventive steps to stop the spread of fires was a but-for cause of the plaintiff's loss. Answer choice C is incorrect. At earlier periods of the common law, cases held landowners strictly liable for the spread of fire. Today, these cases are evaluated under negligence standards. Also, the modern interpretation of Rylands v. Fletcher limits the precedent to a category of "abnormally dangerous activities." There is no indication that the defendant here engaged in an abnormally dangerous activity.

Question 6657 Jumping Land is an indoor playground full of giant inflatable houses and slides upon which children can play and bounce. Large, electric air machines blow air through tubes connected to the inflatables to keep them filled. The managers of Jumping Land do a safety check of the inflatables every morning, and they also inspect the air machines to make sure that the electric motors are running correctly and that the air tubes are securely attached. The air machines are kept behind childproof gates and the owners posted signs warning that the air machines are dangerous. However, to keep them from being an eyesore, the air machines were painted in bright colors so that they fit into the festive motif of Jumping Land. A child happily jumped in the bouncy houses for a while, but eventually he got bored, and asked his father if he could take a closer look at the air machines because he enjoyed learning about how motors worked. The father acquiesced because he would be there to supervise, and unlocked the gate so his son could look at the air machine's motor. When the son got close to the air machine, the air tube suddenly disconnected and whipped the boy in the face, causing permanent damage to one of his eyes. The child's father filed a claim against Jumping Land on his son's behalf for negligence, and Jumping Land responded that it was not liable because the air machines were behind childproof gates and warning signs had been posted. Under the traditional approach regarding invitees and trespassers, is the child's father likely to be successful in the suit against Jumping Land? Answers: Yes, because the air machines used to fill the inflatables were abnormally dangerous. Yes, because the air machines painted in bright colors were an attractive nuisance. No, because the child went into an area from which he was prohibited to enter. No, because the air machine was bounded by a childproof gate and warning signs.

Answer choice D is correct. A land possessor owes an invitee the duty of reasonable care, including the duty to use reasonable care to inspect the property, discover unreasonably dangerous conditions, and protect the invitee from them. Here, the child was an invitee, and Jumping Land owed him a duty of reasonable care to inspect the facility and protect the child from unreasonably dangerous conditions. Jumping Land satisfied this duty as evidenced by the daily safety inspections. However, in this case, the child was a trespasser who exceeded the scope of the invitation at the time of the injury because his father opened the childproof gate for him. The duty of reasonable care does not extend to trespassers. However, because the brightly painted air machines would likely qualify as an attractive nuisance, Jumping Land had a duty to exercise reasonable care in protecting the child from unreasonably dangerous conditions. It satisfied this duty as evidenced by the childproof gates and warning signs surrounding the air machines. Answer choice A is incorrect because the air machines are not abnormally dangerous. Abnormally dangerous means that an activity creates a foreseeable and highly significant risk of physical harm even when reasonable care is exercised, and the activity is not commonly engaged in. In this case, the use of the air machines does not create a foreseeable and highly significant risk of physical harm even when reasonable care is exercised. By performing daily inspections and gating the air machines, as well as posting warning signs, Jumping Land exercised reasonable care which minimized any foreseeable risk of physical harm. Thus, the air machines do not fit the definition of abnormally dangerous. Answer choice B is incorrect because although the brightly painted air machines were an attractive nuisance, Jumping Land exercised reasonable care in protecting the children from harm. Answer choice C is incorrect because the brightly painted air machines were an attractive nuisance, and therefore Jumping Land still owed the child, a trespasser, a duty of reasonable care.

Question 6112 The owner of a pumpkin patch allowed a group of schoolchildren and their teacher to enter the patch to pick their own pumpkins free of charge as part of a class trip. The teacher was on the property with the students to pick his pumpkin when a snake bit him while he was reaching down into the patch. Nearby, the owner had posted a large sign which read, "Danger: Reptiles and rodents present in the patch - exercise caution when picking pumpkins." The teacher brought a suit against the owner to recover damages for his injury. Is the owner likely to be held liable? Answers: Yes, under a strict liability theory based on the inherently dangerous nature of the snake. Yes, because the owner allowed the group onto his land. No, because the owner did not willfully or wantonly cause the injury. No, because the owner warned of the possible hazards.

Answer choice D is correct. A licensee is someone who enters the land of another with the express or implied permission of the land possessor. The land possessor has a duty to either correct or warn a licensee of concealed dangers that are known to the land possessor. The land possessor must also refrain from willful or wanton misconduct. Here, the teacher is a licensee because the owner permitted the teacher and schoolchildren to enter the land to pick their own pumpkins. The owner's duty, therefore, was to not willfully or wantonly cause injury to the schoolchildren and the teacher and to warn them of any known dangers. The owner did not willfully or wantonly cause the teacher's injury. Furthermore, the owner warned the schoolchildren and the teacher by posting a clearly visible sign regarding the presence of reptiles and rodents in the patch. Thus, the owner will not be held liable. Answer choice A is incorrect because strict liability for animals refers to when the owner knowingly keeps the animal on his property (e.g., as a pet). Answer choice B is incorrect because the mere fact that the owner allowed the group on his land is not enough to establish liability. For liability to attach, the owner must have breached a duty to the group. Here, the owner did not breach the duty owed to the group as licensees. Answer choice C is incorrect because while the owner did refrain from willful or wanton misconduct, the owner also had the obligation to warn of known dangers. Thus, answer choice D is the better answer.

Question 7200 A local improvisation troupe organized a "flash mob" in a city park. A "flash mob" is a group of people who agree to assemble quickly and unexpectedly in a public place at a predetermined time, often to perform a skit or choreographed performance, and then to quickly disperse. The troupe choreographed a dance number to be performed in a tight circle around a random pedestrian in the park who they planned to lift onto their shoulders. They also wore matching red shirts that were unobtrusive enough to blend into a crowd, but coordinated enough to make their performance more visually striking once they gathered in one place. After agreeing to perform with the first pedestrian to walk over a chalk circle they drew on the park's sidewalk, the troupe dispersed throughout the park. When an elderly woman walked across the chalk circle, the troupe members charged at her to form their circle. The woman saw many young people running at her in matching red shirts. Terrified that she was being attacked by a gang, the woman fainted, breaking her hip in the fall. If the elderly woman sues the improvisation troupe for assault, is she likely to prevail? Answers: No, because an ordinary person would not conclude that the troupe's conduct exceeded the possible limits of human decency. No, because the troupe only intended to put on an entertaining performance. Yes, because the woman suffered actual physical injury as a result of the troupe's act. Yes, because the troupe intended to lift the woman onto their shoulders without her consent.

Answer choice D is correct. An assault is the plaintiff's reasonable apprehension of an imminent harmful or offensive bodily contact caused by the defendant's action or threat with the intent to cause either the apprehension of such contact or the contact itself. Here, the troupe intended to surround the woman and lift her onto their shoulders without warning or consent while they danced. This is likely to be considered an offensive bodily contact; the woman was terrified that she was being attacked by a gang, and the troupe did not obtain her consent before they intended to lift her onto their shoulders. Therefore, the troupe had the requisite intent to be found liable for a tortious assault, and the woman is likely to prevail in her action. Answer choice A is incorrect because it states the standard for finding extreme or outrageous conduct in an action for the intentional infliction of emotional distress. Meeting this standard is not a requirement of a successful action for assault. Answer choice B is incorrect because the troupe can be found liable for a tortious assault even if they did not intend to harm or frighten the woman. It is enough that they intended to cause an offensive and nonconsensual physical bodily contact. Answer choice C is incorrect because, although the woman's physical injuries may be relevant to the damages she may recover, no proof of actual damages is required to succeed in an action for tortious assault. Therefore, this point is irrelevant to her likelihood of succeeding in her action.

Question 1318 A customer pledged a stock certificate to a bank as security for a loan. A year later, when the customer fully repaid the loan, the bank refused the customer's demand to return the stock certificate because the officer dealing with the loan had the mistaken belief that there was still a balance due. No one at the bank reviewed the records until two months later, at which time the error was discovered. The bank then offered to return the stock certificate. However, the customer refused to accept it. At the time the customer pledged the certificate, the shares were worth $10,000; at the time the customer repaid the loan, the shares were worth $20,000; and at the time the bank offered to return the certificate, the shares were worth $5,000. If the customer brings an action against the bank based on conversion, how much, if anything, should the customer recover? Answers: Nothing, because the bank lawfully came into possession of the certificate. $5,000, because that was the value of the shares when the customer refused to accept the certificate back. $10,000, because that was the value of the shares when the bank came into possession of the certificate. $20,000, because that was the value of the shares when the customer was entitled to the return of the certificate.

Answer choice D is correct. Conversion is equivalent to a forced sale of the chattel to the defendant, who is liable for the full value of the chattel at the time of the tort. The tort occurred when the bank refused to relinquish the stock certificate in response to the customer's lawful demand, and at that time the shares were worth $20,000. Answer choice A is incorrect. Someone who refuses to surrender a chattel to another person who is entitled to its immediate possession is liable for conversion even if the one holding the chattel originally came into possession lawfully. The bank is liable to the customer for the value of the shares at the time the bank refused the customer's lawful demand for return of the certificate - $20,000. Answer choices B and C are incorrect because the defendant is liable for the full value of the chattel at the time of the tort - when the bank refused to relinquish the stock certificate in response to the customer's lawful demand, and at that time the shares were worth $20,000.

Question 6655 A mother and her two children shopped and browsed at a large discount store for approximately an hour before paying for their purchases. As they left the store, two store employees stepped in front of the family and blocked their path to the exit. The employees told the mother that her children were suspected of shoplifting on a prior occasion, that they had called the police, and that the children would go to juvenile hall for the rest of their childhood based upon the videotape evidence they had of the children stealing store items. In reality, the employees only called the store security officer, and they did not have any videotape surveillance. After a few minutes, the security officer arrived and told the employees that the children were not the kids that had been caught shoplifting earlier. The employees apologized for their mistake, and the family left the store. The mother brings a claim for false imprisonment against the employees, and they move for summary judgment. Is the court likely to allow the mother's claim to proceed? Answers: No, because the employees only detained the mother and her children for a few minutes. No, because the employees' detention was a reasonable and valid use of their authority. Yes, because the children were cleared of any wrongdoing and the employees made a mistake. Yes, because a threat or claim of lawful authority to detain is sufficient to show confinement.

Answer choice D is correct. False imprisonment results when a person acts intending to confine or restrain another within boundaries fixed by the actor, those actions directly or indirectly result in such confinement, and the other is conscious of the confinement or is harmed by it. The employees' threat of calling the police was sufficient to show confinement. Further, while the shopkeeper's privilege allows for a reasonable (in both time and manner) detention of suspected shoplifters to give them time to investigate. This reasonableness is based on the totality of the circumstances. In this case, the employees threaten the children with juvenile hall, lie about the video surveillance, and lie about having called the police, which strongly suggests that they are acting under color of law by holding the family until the police arrive. It is likely that a jury would conclude that threatening children while lying about having evidence and having called the police was extreme and not reasonable. Answer choice A is incorrect because the length of time of the confinement or restraint is immaterial, except as to the determination of the extent of damages. Answer choice B is incorrect because the detention was not a reasonable and valid use of their authority under the protection of the shopkeeper's privilege. Threatening to call the police and claiming to have videotape surveillance of the children stealing was not reasonable. Answer choice C is incorrect because a guilty shoplifter can still sue for false imprisonment if the detention was unreasonable.

Question 6116 In a contributory negligence jurisdiction, Carla was riding her valuable racing bike and ran over a nail in the street. The bike developed a flat tire and Carla negligently left it in the middle of a street while she went into her garage to get something to patch the tire. Deborah was driving down Carla's street, saw the bike in the road, but refused to swerve to avoid the bike. Deborah hit the bike, damaging her own car and destroying Carla's racing bike. Does Carla have a valid claim against Deborah for negligence? Answers: No, because the accident occurred in a contributory negligence jurisdiction. No, because Carla was a direct cause of her own damages. Yes, because of the theory of comparative fault. Yes, because of the last clear chance doctrine.

Answer choice D is correct. In contributory negligence jurisdictions, a plaintiff may mitigate the legal consequences of her own contributory negligence if she proves that the defendant had the last clear chance to avoid injuring the plaintiff but failed to do so. Here, Carla was contributorily negligent, but Deborah had the last clear chance to avoid the accident. Answer choice A is incorrect. While the accident did occur in a contributory negligence jurisdiction, the doctrine of last clear chance applies to allow Carla to make a claim against Deborah despite her own negligence in leaving the bike in the street. Answer choice B is incorrect. While Carla helped to cause her own damages by leaving her bike in the street, the doctrine of last clear chance applies to allow a claim against Deborah, who saw the bike and refused to swerve. She had the last clear chance to avoid the accident. Answer choice C is incorrect. The facts indicate that the jurisdiction is a contributory negligence jurisdiction, not a comparative-fault jurisdiction. Accordingly, comparative fault does not apply. Instead, the reason Carla has a claim against Deborah for negligence is the doctrine of last clear chance, as explained above.

Question 1287 When a tire of a motorist's car suffered a blowout, the car rolled over and the motorist was badly injured. Vehicles made by the manufacturer of the motorist's car have been found to be negligently designed, making them dangerously prone to rolling over when they suffer blowouts. A truck driver who was driving behind the motorist when the accident occurred stopped to help. Rescue vehicles promptly arrived, and the truck driver walked along the side of the road to return to his truck. As he approached his truck, he was struck and injured by a speeding car. The truck driver has sued the manufacturer of the injured motorist's car. Is the truck driver likely to prevail in a suit against the car manufacturer? Answers: No, because the car manufacturer's negligence was not the proximate cause of the truck driver's injuries. No, because the truck driver assumed the risk of injury when he undertook to help the motorist. Yes, because it is foreseeable that injuries can result from rollovers. Yes, because the car manufacturer's negligence caused the dangerous situation that invited the rescue by the truck driver.

Answer choice D is correct. It is well established that injury to rescuers is sufficiently foreseeable to support proximate cause. As Cardozo said, "Danger invites rescue." Answer choice A is incorrect. Even though the chain of causation here is lengthy, it is well established that injury to rescuers is sufficiently foreseeable to support proximate cause. Answer choice B is incorrect. The danger to pedestrians of speeding cars on a highway is foreseeable. But the defense of assumption of risk requires more specific knowledge of the risk that is run, as well as either agreement to take responsibility for the risk or unreasonable risk-taking on the part of the plaintiff. There is no indication here that the truck driver agreed to take full responsibility for the risk of being struck by a speeding car or unreasonably exposed himself to that risk. Answer choice C is incorrect. In order to establish proximate cause, something more specific than "injuries" broadly defined must be foreseeable. The kind of injury that had to be foreseeable to the manufacturer in this case is that someone engaged in a rescue attempt after a car accident might be struck by a speeding vehicle on the road.

Question 1325 A retail company hires an accountant to conduct an independent audit of its books and records. The accountant is aware that his opinion may be relied upon by the public at large in assessing the financial health of the company and is specifically informed that the company intends to use the opinion to obtain goods from a supplier. Due to an inadvertent miscalculation, the accountant erroneously provides the company with a favorable, unqualified opinion. The company gives the accountant's opinion to the supplier. Impressed by the opinion, the supplier decides to purchase the company instead of supplying goods to the company. Subsequently, after learning of the true condition of the retailer, the supplier sells the company at a loss. The supplier brings an action against the accountant for negligent misrepresentation. Who will prevail? Answers: The supplier, because the supplier relied on the accountant's opinion. The supplier, because the accountant was aware that the supplier was the intended user of the opinion. The accountant, because the accountant's false opinion was based on an inadvertent mistake. The accountant, because the accountant's liability does not extend to the supplier's actual use of the accountant's opinion.

Answer choice D is correct. The elements of negligent misrepresentation are: (i) the defendant provided false information, (ii) as a result of the defendant's negligence, (iii) during the course of his business or profession, (iv) causing the plaintiff to justifiably rely upon the information, and (v) the plaintiff either is in a contractual relationship with the defendant or is a third party known by the defendant as one for whose benefit the information is supplied. Here, although the elements of negligent misrepresentation are otherwise met, the accountant is not liable if the third party's use of the information is of a different character than the use for which the accountant provided the information. That is the case here because the accountant's opinion was prepared to aid the supplier in determining whether to sell goods to the retail company rather than to purchase the company. Answer choice A is incorrect because, while the supplier's reliance on the accountant's opinion is necessary to establish negligent misrepresentation, it alone is not sufficient for the reason discussed with respect to answer choice D. Answer choice B is incorrect because, even though the accountant was aware that the supplier was the intended user of the opinion, the answer choice does not allow for the fact that liability is nevertheless limited. Answer choice C is incorrect because the action is for negligent misrepresentation, not intentional misrepresentation. Consequently, the accountant's negligence can serve as a basis for liability.

Question 6548 A woman hired a personal trainer to help her lose weight. In addition to enforcing a rigorous workout regimen, the trainer gave the woman appetite suppressant pills that the trainer claimed to have formulated himself. The trainer told her that the pills contained a combination of natural herbs that would safely curb her food cravings. The trainer did not charge the woman for the pills. The woman took the pills for a week. Although she stopped having food cravings, she also started to feel dizzy and had a hard time breathing. The woman showed the pills to a pharmacist, who identified them as store-bought pills containing a drug that suppressed one's appetite, but that had potentially dangerous side effects such as dizziness, headaches, or respiratory difficulty. The woman immediately stopped taking the pills and within a day she stopped feeling dizzy and having difficulty breathing. If the woman files suit against the trainer for intentional misrepresentation, will she succeed? Answers: Yes, because she suffered dangerous side effects when she took the pills. Yes, because she relied on the trainer's false assertion that the pills were safe. No, because the pills helped suppress the woman's appetite. No, because she suffered no actual economic loss.

Answer choice D is correct. To recover for intentional misrepresentation, the plaintiff must establish a misrepresentation by the defendant, scienter, intent to induce the plaintiff's reliance, justifiable reliance by the plaintiff, and actual economic damages. Here, the woman relied on the trainer's false assertions regarding the pills with the intent to induce her reliance on his statements, and the woman justifiably relied upon his false assertions. However, the woman did not suffer any pecuniary loss since she did not pay for the pills. Thus, while she likely has some tortious claim against the trainer, she will not succeed on a claim for intentional misrepresentation. Answer choice A is incorrect. Although the woman experienced side effects from taking the drugs, she did not suffer any pecuniary loss because she did not pay for pills. Answer choice B is incorrect because although she relied on the trainer's false assertions, she did not suffer any pecuniary loss. Answer choice C is incorrect. Although the pills helped to suppress the woman's appetite, this would not protect the trainer from liability if the woman had also suffered actual economic losses.

Question 6175 A chef agreed to purchase a rare bottle of wine from a wine collector's personal collection. The wine collector immediately shipped the bottle overnight to the chef through a delivery company that specialized in shipping wine. The next evening, the chef served the bottle of wine, but the wine had been spoiled, making him and his guests ill and rendering the wine worthless. The chef sued both the wine collector and the delivery company in negligence to recover damages. At trial, the chef introduced a wine expert that testified that the bottle of wine had been subjected to high temperatures that were inappropriate for preserving wine before the bottle reached the chef. However, the expert was unable to identify which defendant had improperly stored the wine; none of the other evidence presented by the chef could prove this, either, nor could any evidence presented allow the jury to allocate the amount of damages between the two defendants. At the close of evidence, the defendants moved for a directed verdict. The jurisdiction follows the doctrine of joint and several liability. Is the defendants' motion for a directed verdict likely to succeed? Answers: No, because the wine collector and the delivery company contributed to an indivisible harm. No, because the wine would not have been damaged if not for the negligence of at least one of the two defendants. Yes, because the chef could not present evidence to allow the jury to allocate the damages between the two defendants. Yes, because the chef cannot prove that both of the defendants acted negligently.

Answer choice D is correct. Under the theory of alternative causation, if the plaintiff's harm was caused by a small number of defendants (usually between two and five), all of those defendants acted tortiously, and all of them are before the court, the court may shift the burden of proof to the defendants to prove that his conduct was not the cause of the plaintiff's harm. However, the initial burden is on the plaintiff is to prove that all of the defendants did, in fact, act negligently. In this case, while the chef can prove that someone acted negligently by subjecting the wine to an unacceptably high temperature, he cannot prove that both the collector and the delivery driver actually acted negligently. Accordingly, the chef has not established his prima facie case for negligence. Answer choice A is incorrect because joint and several liability applies when the tortious acts of two or more tortfeasors combine to produce an indivisible harm. Here, the chef has not presented evidence that both parties committed tortious acts. The chef must first establish the inference of negligence by both parties before this is an effective argument. Answer choice B is incorrect. Res ipsa loquitor can be used against multiple defendants if the plaintiff is unable to prove who among them was negligent, but this is generally used against parties who are at least acting in concert, such as in a surgical setting in a medical malpractice action, or occasionally in products liability actions. Typically, to hold two or more defendants responsible for one harm, there must be some evidence of negligence on the part of both defendants. Answer choice C is incorrect because, if two or more defendants who are present before the court have both acted tortiously, the court may shift the burden of proof to the individual defendants to prove that his conduct was not the cause of the plaintiff's harm. Generally, when defendants are jointly and severally liable to a plaintiff, the plaintiff can recover the whole amount from any defendant, and it is the defendants' responsibility, not the plaintiff's, to establish who should be responsible for what percentage of damages.

Question 5997 A plaintiff owns and operates a food truck. The defendant, a construction worker, negligently backed some heavy machinery into the plaintiff's food truck, damaging it. The impact caused the plaintiff, who was in the food truck at the time, to fall over and hit his head on the counter, rendering him temporarily unconscious. While the plaintiff was out, the defendant watched as a thief entered the food truck and rifled through the cash register, stole several hundred dollars, and disappeared. Although she could have easily and safely thwarted the theft, the defendant took no action. The plaintiff brought an action against the defendant. In addition to claims for the injury to himself and the damage to his food truck, the plaintiff is seeking to recover the money that was stolen, since the plaintiff has been unable to identify the thief. Which of the following best describes the likely outcome of the plaintiff's claim to recover the stolen money from the defendant? Answers: Denied, because the money was taken by a thief. Denied, because there is no duty to protect a stranger from criminal activity. Allowed, because the plaintiff is unable to recover from the thief. Allowed, because the defendant's conduct had left the plaintiff vulnerable to the thief.

Answer choice D is correct. While a person is generally not liable for the criminal acts of another, and has no duty to prevent such acts, a person who places another in peril is under a duty to exercise reasonable care to prevent further harm by rendering care or aid. Here, the defendant's negligent conduct rendered the plaintiff unconscious and placed him at the mercy of the thief. Consequently, the defendant had a duty to exercise reasonable care to prevent the theft. Since she did not, she is liable to the plaintiff for his loss of property (i.e., cash) that occurred as a result. Answer choice A is incorrect because, although the thief is certainly liable to the plaintiff for conversion, since the defendant placed the plaintiff in peril, she was under a duty to exercise reasonable care to prevent further harm by rendering care or aid, which she failed to do. Answer choice B is incorrect because, while generally there is no affirmative duty to act, such as to protect a stranger from criminal activity, since the defendant placed the plaintiff in peril, she was under a duty to exercise reasonable care to prevent further harm by rendering care or aid, which she failed to do. Answer choice C is incorrect because the defendant's liability to the plaintiff is independent of the thief's liability to the plaintiff. Even if the plaintiff was aware of the identity of the thief and could bring suit against the thief, the plaintiff can elect to pursue his claim for the stolen cash against the defendant instead.


Conjuntos de estudio relacionados

REVIEW OF THE ACCOUNTING PROCESS - Theories Part IV

View Set